Yahoo Answers is shutting down on May 4th, 2021 (Eastern Time) and beginning April 20th, 2021 (Eastern Time) the Yahoo Answers website will be in read-only mode. There will be no changes to other Yahoo properties or services, or your Yahoo account. You can find more information about the Yahoo Answers shutdown and how to download your data on this help page.

IVP using Laplace transform ?

y' + y = e^4t; y(0) = 2

I think I understand the proceedure, but what do I do with y(0) = 2? So the process as I understand it:

L(y') + L(y) + L(2) = L(e^4t)

sY(s) + Y(s) + (1/s) = 1/(s-a) is this correct?

Thanks

Update:

Thank you, that answers my question. I tried it a second time with the y(0)=2 and was up to the partial fraction part. Looks like I was headed in the right direction.

sp33...I'm getting used to those, they're not as scary anymore.

2 Answers

Relevance
  • 1 decade ago
    Favorite Answer

    For laplace transforms remeber that:

    y' -> sY(s) -y(0)

    Here y(0) is the value of y(t) when t=0.

    Which becomes:

    sY(s) -y(0) + Y(s) = 1/(s-4)

    Y(s)[s+1]= 1/(s+4) + 2

    Y(s) = 1/[(s-4)*(s+1)] + 2/(s+1)

    Now you have to find partial frations of:

    1/[(s-4)*(s+1)]

    becomes:

    (-1/5)*(1/(s-4)) + (1/5)*(1/(s+1))

    So:

    Y(s) = (-1/5)*(1/(s-4)) + (1/5)*(1/(s+1)) + 2/(s+1)

    With some cleaning up:

    Y(s) = (-1/5)*(1/(s-4)) + (11/5)*(1/(s+1))

    Now you can find the inverse:

    y(t) = (11/5)*e^-t + (-1/5)*e^4t

    Partial fractions.. learn them love them breathe them.. Especially for laplace transforms.

    Source(s): http://math.fullerton.edu/mathews/c2003/LaplaceDif... http://video.google.com/videoplay?docid=9770642651... Electrical Engineering in University.. lots of laplace transforms
  • 1 decade ago

    The catch is in L(y')

    Using Integration by Parts, you can show that L(y^(n)) (where n is the differentiation level, not an exponent) is equal to the series:

    sY(s) - y^(n-1)(0) - s*y^(n-2)(0) - s^2*y^(n-3)(0) - ... - s^(n-2)*y'() - s^(n-1)*y(0)

    Basically, this means that L(y') = sY(s) - y(0). Using this rule, your equation becomes:

    L(y' + y) = L(e^4t)

    sY(s) - 2 + Y(s) = 1/(s-4)

    And no - you can't just throw y(0) onto one side of the equation to get it in there. This completely destroys the equation, and as my Professor would say, "All hope is lost."

Still have questions? Get your answers by asking now.